[Math] proof of the AM-GM Inequality via Cauchy’s Inequality? Or vice-versa

a.m.-g.m.-inequalityalternative-proofcauchy-schwarz-inequalityinequalityreal-analysis

The title says it all. There is already a list exhausting proofs of the AM-GM inequality here; however, none via Cauchy's Inequality.

Why do I care? Recreation. But more than that, proving inequalities via each other is one of the predominant past-times in the theory of inequalities. I'm slightly surprised this isn't a common proof out there. And a proof via this route sounds like it would demonstrate either extreme algebraic subtlety and cleverness or give rise to a new technique in proving inequalities. Venerable ends, each of them.

Undoubtedly, Cauchy's Inequality and the AM-GM Inequality are connected in some fashion. Jensen's Inequality can prove both of them. However, it is interesting to note that difference in the proofs via Jensen's. The proof of Cauchy via Jensen relies on the boring observation that $x^2$ (an algebraic function) is strictly convex followed by algebraic hyjinx and cleverness in choosing weights and evaluation points. On the other hand, the proof of the AM-GM Inequality via Jensen relies more on the foresight or prescience of using the exponential function (a transcendental function) with no funny business involving the weights or evaluation points.

Both can be proved by Cauchy Induction, although the base cases differ in that they rely on
$$\sqrt{xy}\leq \frac{x+y}{2}\quad\text{and}\quad(x_1y_1+x_2y_2)^2\leq (x_1^2+x_2^2)(y_1^2+y_2^2)$$
respectively. Both can be proved by normalization, although the exact reductions they are proved from differ. Cauchy relies on
$$\sum_{k=1}^n |x_ky_k|\leq 1\quad\text{whenever}\quad\sum_{k=1}^nx_k^2=1=\sum_{k=1}^ny_k^2$$
while the AM-GM relies on
$$\alpha_1+\alpha_2+\cdots+\alpha_n\geq n\quad\text{whenever}\quad \alpha_1\alpha_2\cdots\alpha_n=1\,.$$

Frequently, one can discover that one inequality can be proved from another if they have similar sets where equality can hold. For example, Minkowski's inequality is frequently proven with Holder's Inequality and the triangle inequality (an inequality which is typically too coarse for most things). However, the triangle inequality and Minkowski's Inequality share very similar sets where equality can happen. This makes me focus on the points where equality can hold in either inequality.

Let $E=(0,\infty)$. Then the set where equality can hold in Cauchy's inequality is
$$\{(x;y)\in E^{2n}:\text{ there is a }\lambda>0\text{ such that }x=\lambda y\}$$
while the set where equality can hold in the AM-GM inequality is
$$\{x\in E^n: \text{there is a }\lambda>0 \text{ such that } x=\lambda(1, 1, \ldots, 1)\}\,.$$
These are extremely different sets and hint that a proof between the two might be difficult to find. However, I doubt it's impossible. There will undoubtedly be trickery involved in passing between these different dimensions.

To lend some credence that it could be possible, Cauchy's Inequality is strong enough to prove the AM-HM Inequality for it is clear that
$$n^2\leq \left(\sum x_k\right)\left(\sum 1/x_k\right)\,.$$
This proof requires a clever choice of '$y$' in terms of '$x$' to pass to the lower dimension. Thus, it might be possible to find a function $f$ such that if $x=(x_1, x_2, \ldots, x_n)$ and $y=(f(x_1), f(x_2), \ldots, f(x_n))$ then Cauchy's Inequality could result in the AM-GM inequality.

Best Answer

A nice proof can be found in chapter 2 of The Cauchy-Schwarz Master Class by J.M. Steele. It is stated in form of some problems starting with:

Problem 2.1: Show that for every sequence of nonnegative real numbers $a_1,a_2,\ldots,a_n$ one has \begin{align*} (a_1a_2\cdots a_n)^{1/n}\leq \frac{a_1+a_2+\cdots a_n}{n}\tag{1} \end{align*}

The proof of this and the following problems is done in some steps.

Step 1: $n=2^k, k\geq 1$

From the Cauchy-Schwarz inequality \begin{align*} \sqrt{a_1a_2}\leq \frac{a_1+a_2}{2}\tag{2} \end{align*}

which is (1) with $n=2$ we can go on one step to $n=4$ by applying (2) twice \begin{align*} (a_1a_2a_3a_4)^{\frac{1}{4}}\leq \frac{(a_1a_2)^{\frac{1}{2}}+(a_3a_4)^{\frac{1}{2}}}{2} \leq \frac{a_1+a_2+a_3+a_4}{4} \end{align*}

This new bound can be used to prove (1) for $n=8$ \begin{align*} (a_1a_2\cdots a_8)^{\frac{1}{8}}\leq \frac{(a_1a_2a_3a_4)^{\frac{1}{4}}+(a_5a_6a_7a_8)^{\frac{1}{4}}}{2} \leq \frac{a_1+a_2+\cdots+a_8}{8} \end{align*}

Iteratively going on this way we can show AM-GM for $n=2^k,k\geq 1$.

\begin{align*} \color{blue}{(a_1a_2\cdots a_{2^k})^{\frac{1}{2^k}}\leq\frac{a_1+a_2+\cdots+a_{2^k}}{2^k}} \end{align*}

Step 2: $2^{k-1}<n<2^k$

The next step is to fill the gaps between $2^{k-1}$ and $2^k$. This is done by taking a value $n<2^k$, considering $\alpha_i=a_i$ for $1\leq i\leq n$ and setting \begin{align*} \alpha_i=\frac{a_1+a_2+\cdots+a_n}{n}\equiv A\qquad\qquad n<i\leq 2^k \end{align*}

The sequence $\{a_i:1\leq i\leq n\}$ is transformed to a sequence $\{\alpha_i:1\leq i\leq 2^k\}$ padded with $2^k-n$ copies of the average $A$ and it can be shown \begin{align*} \color{blue}{(a_1a_2\cdots a_n)^{1/n}\leq \frac{a_1+a_2+\cdots+a_n}{n}} \end{align*}

Step 3: rational numbers

Based upon step (2) the AM-GM is shown for non-negative rational numbers $p_1,p_2,\ldots,p_n$ that sum to one. \begin{align*} \color{blue}{a_1^{p_1}a_2^{p_2}\cdots a_n^{p_n}\leq p_1a_1+p_2a_2+\cdots +p_na_n} \end{align*}

Step 4: real numbers

Based upon step (3) the AM-GM is shown for non-negative real numbers $p_1,p_2,\ldots,p_n$ that sum to one by taking limits.

\begin{align*} \color{blue}{a_1^{p_1}a_2^{p_2}\cdots a_n^{p_n}\leq p_1a_1+p_2a_2+\cdots +p_na_n} \end{align*}

Note: All these steps are some kind of bootstrapping based upon the step before and starting with the Cauchy-Schwarz inequality.

Related Question